2023 AMC 12B Problems/Problem 20

Revision as of 18:37, 15 November 2023 by Professorchenedu (talk | contribs) (Created page with "==Solution== Denote by <math>A_i</math> the position after the <math>i</math>th jump. Thus, to fall into the region centered at <math>A_0</math> and with radius 1, <math>\ang...")
(diff) ← Older revision | Latest revision (diff) | Newer revision → (diff)

Solution

Denote by $A_i$ the position after the $i$th jump. Thus, to fall into the region centered at $A_0$ and with radius 1, $\angle A_2 A_1 A_0 < 2 \arcsin \frac{1/2}{2} = 2 \arcsin \frac{1}{4}$.

Therefore, the probability is $$ (Error compiling LaTeX. Unknown error_msg) \[ \frac{2 \cdot 2 \arcsin \frac{1}{4}}{2 \pi} = \boxed{\textbf{(E) $\frac{2 \arcsin \frac{1}{4}}{\pi}$}}. \] $$ (Error compiling LaTeX. Unknown error_msg)

~Steven Chen (Professor Chen Education Palace, www.professorchenedu.com)